Irreducibility and finite fields

Click For Summary
The discussion revolves around solving parts i) and iii) of an exam question related to irreducibility and finite fields. For part i), the focus is on proving that the polynomial g(X) has no roots in the finite field k, specifically by testing the values 0 and 1, which leads to the conclusion that g(X) is irreducible if neither evaluates to zero. In part iii), the discussion clarifies that every element in K is separable over k, but confusion arises regarding the minimal polynomial having repeated roots, suggesting it may be inseparable. The participants also clarify the structure of the finite field k, noting it includes elements like T and 1+T. The conversation concludes with a realization of a previous oversight regarding the extended element T.
Firepanda
Messages
425
Reaction score
0
33cou48.jpg


Trying to do i) and iii) on this past exam paper

For part i) I'm pretty stumped

I've said that the possible roots of the polynomial are +- all the factors of T

In particular rt(T) needs to be a factor of T but this can't be possible?

Doesn't sound too good but its the best I've got.

Part iii) I know this means that every element in K is seperable over k, i.e that the minimal polynomials of elements in K are seperable, where they have no repeated roots.

Not sure how to apply this though..

Maybe K = k(rt(T))

so the minimal polynomial of K is X^2 - T which has repeated root rt(T) so it is inseperable?
 
Last edited:
Physics news on Phys.org
For part i:

[STRIKE]Because this is a finite field, you must show that the polynomial g(X) has no roots in k. Because k is a finite field of 2 elements, you can just try plugging in 0 and 1 because k = {0,1}.

So g(X) is irreducible if and only if 0^2-T is not equal to zero and 1^2-T is not equal to zero. Since T is an extended element onto k and is therefore not equal to either 0 or 1, which is what we need to make the above two equations equal to zero, I think that we can safely draw our conclusion. What do you think?[/STRIKE]

edit: I'm dumb. i'll rethink this.
 
micaele said:
For part i:

Because this is a finite field, you must show that the polynomial g(X) has no roots in k. Because k is a finite field of 2 elements, you can just try plugging in 0 and 1 because k = {0,1}.

So g(X) is irreducible if and only if 0^2-T is not equal to zero and 1^2-T is not equal to zero. Since T is an extended element onto k, I think we can safely draw our conclusion. What do you think?

isn't k = {0, 1, T, 1+T} since k = {a + bT | a, b in {0,1}}

Otherwise I see what you mean. Thanks for the reply
 
yeah you're completely right and i noticed my mistake right after i posted that. i overlooked the whole extended element thing. my bad.
 
bump for confirmation
 
Question: A clock's minute hand has length 4 and its hour hand has length 3. What is the distance between the tips at the moment when it is increasing most rapidly?(Putnam Exam Question) Answer: Making assumption that both the hands moves at constant angular velocities, the answer is ## \sqrt{7} .## But don't you think this assumption is somewhat doubtful and wrong?

Similar threads

  • · Replies 1 ·
Replies
1
Views
2K
  • · Replies 1 ·
Replies
1
Views
2K
  • · Replies 28 ·
Replies
28
Views
5K
Replies
6
Views
2K
  • · Replies 6 ·
Replies
6
Views
3K
  • · Replies 3 ·
Replies
3
Views
2K
  • · Replies 1 ·
Replies
1
Views
2K
  • · Replies 2 ·
Replies
2
Views
1K
  • · Replies 12 ·
Replies
12
Views
2K
  • · Replies 1 ·
Replies
1
Views
1K